LSAT and Law School Admissions Forum

Get expert LSAT preparation and law school admissions advice from PowerScore Test Preparation.

User avatar
 Dave Killoran
PowerScore Staff
  • PowerScore Staff
  • Posts: 5853
  • Joined: Mar 25, 2011
|
#44136
Complete Question Explanation
(The complete setup for this game can be found here: lsat/viewtopic.php?t=16592)

The correct answer choice is (E)

Because there are currently six assistants, and one must be promoted this year, at most there will be five assistants next year. Because at least one assistant must be promoted next year, at most there will be four assistants after next year’s review. It follows that answer choice (E) is correct.
 gracekim
  • Posts: 2
  • Joined: Sep 28, 2012
|
#5811
I was looking at this game today and I'm not sure if I'm understanding the questions right. I saw the explanation for question 22 in another post but I couldn't figure out 23. If anyone can help me out it would be greatly appreciated. Thank you!
 Jon Denning
PowerScore Staff
  • PowerScore Staff
  • Posts: 904
  • Joined: Apr 11, 2011
|
#5864
Hey Grace (Kim?) - thanks for the question. It would help me out if you could be a little more specific about exactly what gave you trouble with the question, but I'll try to add some insight anyway.

23. Since this question is about "next year's review" then we have two reviews to consider (this year's AND next year's). And since, according to the scenario given, at least one assistant is promoted at each review, then at least two of the assistants must have been promoted. So it must be true after two reviews that at most only four of the original six assistants remain, answer choice D.

Hope that helps!
 gracekim
  • Posts: 2
  • Joined: Sep 28, 2012
|
#5872
Thank you for your reply. I reread the problem and I missed a rule! Doh!
User avatar
 lawlandmem
  • Posts: 9
  • Joined: Mar 24, 2021
|
#86554
I'm a bit thrown off by this game... so the rules say that at least 1 assistant and associate must be promoted, but that doesn't mean that only 1 assistant was promoted, right? So how can we assume that only 1 assistant was promoted this year?
 Rachael Wilkenfeld
PowerScore Staff
  • PowerScore Staff
  • Posts: 1358
  • Joined: Dec 15, 2011
|
#86579
We aren't assuming that only one assistant was promoted, lawlandmem, we are following the rule that at least one assistent per year is promoted. That's why our correct answer choice says that there are at most four assistants left. Each year at least one gets promoted. That's a minimum of two assistants out of six that are promoted, leaving at most four assistants. It's possible that two assistants are promoted each year. That's consistent with the rules and the answer choice. There's still at most four assistants.

Hope that helps!
User avatar
 goingslow
  • Posts: 52
  • Joined: Aug 24, 2021
|
#89951
Hi! I have a follow-up question for answer (D). For question #23, after next year's review, could it be true that there were NO assistants? My thought process is that during this year's promotion review, all 6 assistants can be promoted except J. So J is the only one who has to be left behind this year. Next year, maybe J can be promoted, leaving no assistants behind. Am I right in thinking this way?
 Adam Tyson
PowerScore Staff
  • PowerScore Staff
  • Posts: 5153
  • Joined: Apr 14, 2011
|
#89952
You are correct that the maximum number of assistants after next year's review is zero, goingslow! Everyone but J could be promoted this year, and then J would have to be promoted the following year. That disproves answer D!

Get the most out of your LSAT Prep Plus subscription.

Analyze and track your performance with our Testing and Analytics Package.